Is this a linear function? y-8= -8(x-2)

Answers

Answer 1

Answer:

Yes

Step-by-step explanation:

y-8= -8(x-2)  ==> solve for y

y = -8(x - 2) + 8  ==> add 8 on both sides to isolate y

y = (-8 * x - 2(-8)) + 8  ==> distribute -8 to x and -2 using the distribution

                                         property

y = (-8x - (-16)) + 8 ==> simplify

y = (-8x + 16) + 8  ==> subtracting by a negative number is equivalent to

                               adding by a positive number.

y = -8x + 16 + 8

y = -8x + 24  ==> Degree of x in the equation is 1, so the function is linear


Related Questions

A radioactive substance decays according to A=A0e−0.0037t, where A0 is the initial amount and t is the time in years. If A0=740 grams, find the time for the radioactive substance to decay to 373 grams. Round your answer to two decimal places, if necessary.

Answers

The time is taken by the substance to decay, if A radioactive substance decays according to [tex]A = A_o \times e^{-0.0037t}[/tex] Where A₀ is the initial amount and t is the time in years. If A0=740 grams is  185.18 years.

What are radioactive substances?

Atoms in radioactive materials naturally decay. They are capable of emitting gamma radiation, beta radiation, and alpha radiation. They cannot be turned off, so controlling them is more challenging than controlling X-ray sources.

Given:

The initial amount, A₀ = 740 g,

The final amount, A = 373 g,

Calculate the time by using the formula given below,

[tex]A = A_o \times e^{-0.0037t}[/tex]

373 = 740 [tex]\times e^{-0.0037t}[/tex]

[tex]e^{-0.0037t}[/tex] = 373 / 740

[tex]e^{-0.0037t}[/tex] = 0.504

Taking log on both sides,

㏑ 0.504 - ㏑ 0.0037 = t

t = 185.18

Therefore, the time taken by the substance to decay if A radioactive substance decays according to A=A0e−0.0037t, where A0 is the initial amount and t is the time in years. If A0=740 grams is 185.18 years.

To know more about radioactive substances:

https://brainly.com/question/1160651

#SPJ1

how many degrees was the figure rotated; rotate the point (-3,-4) around the origin 180 degrees; rotate the point (7 8) around the origin 90 degrees; rotate the point (7 8) around the origin 90 degrees counterclockwise; if you were to rotate abcd 180; rotate the point (-3,-4) around the origin 180 degrees. what is the image of the point?; if you were to rotate abcd 90 degrees counterclockwise; what is the angle of rotation for this counterclockwise rotation about the origin

Answers

a) 270° counter clockwise rotation of figure 1

b) The image of point (-3,-4) is (3,4).

c) The new image of point (7,8) is (8,-7) after 90° degree clockwise rotation.

d) The new image of point (7,8) is (-8,7) after 90° degree anticlockwise rotation

e) The fig. abcd will moves to second quardent after 90° counter clockwise rotation.

f) The rotation angle of figure is 270° , in the counterclockwise direction about origin.

A rotation is an isometric transformation that turns every point of a figure through a specified angle and direction about a fixed point.

To describe a rotation, you need three things:

Direction (clockwise or counterclcokwise.

Angle in degreesCenter point of rotationRotations About The Origin

90 Degree Rotation:

When rotating a point 90 degrees counterclockwise about the origin of a point A(x,y) becomes A'(-y,x). In other words, switch x and y and make y negative.

180 Degree Rotation :

When we rotating a point 180 degrees counterclockwise about the origin a point A(x,y) it becomes A'(-x,-y). So all we do is make both x and y negative.

We have given that,

a) The given figure is rotated aoround 90° clockwise or 270° counter clockwise.

b) A point (-3,-4) is rotated around the origin 180 degrees , then the new point or position after the rotation is (3,4).

c) Rotate the point (7 , 8) around the origin 90 degrees , the new point or image of (7,8) is ( 8,-7).

d) If a point (7,8) is rotate around the origin 90 degrees counterclockwise then new point is (-8,7).

e) If you were to rotate abcd 90 degrees counterclockwise then the image of figures abcd will enter in quadrent 2nd with all coordinate changes.

f) when a figure is rotated in counterclockwise rotation about the origin the angle of rotation is 270 degrees.

To learn more about Rotation of figure, refer:

https://brainly.com/question/27305576

#SPJ4

64% of all vehicles examined at a certain emissions inspection station pass the inspection. assuming that successive vehicles pass or fail independently of one another, calculate the probability that exactly one of the next three vehicles fail. (give your answer as a decimal number with 3 digits of precision.)

Answers

The probability that exactly one of the next three vehicles fail is 0.737 .

Given :

64% of all vehicles examined at a certain emissions inspection station pass the inspection. assuming that successive vehicles pass or fail independently of one another .

probability that exactly one of the next three vehicles fail is :

P = 1 -  probability that all of the next three vehicles passes .

= 1 - 64 % * 64 % * 64 %

= 1 - 64/100 - 64/100 - 64/100

= 1 - 0.64 * 0.64 * 0.64

= 1 - 0.4096 * 0.64

= 1 - 0.262

= 0.737

Learn more about the probability here:

https://brainly.com/question/11234923

#SPJ4

Which is called vertex?

Answers

A vertex is a point where two or more curves, lines, or edges meet.

In geometry, vertex generally refers to an intersection point or a corner where lines, curves or edges meet. As a consequence of this definition, the point where two lines meet to form an angle called an included angle are considered vertices. Hence, the corners of triangle, polygons and polyhedral are vertices. For example, the lines of square or rectangle intersect at four corners which are called its vertices (which is the plural of vertex). Similarly, the vertex of curve(s) is where they intersect with each other or with its line of symmetry.

Learn more about Vertex:

https://brainly.com/question/15709421

#SPJ4

The T distribution :
A) assumes the population is normally distributed
B) approaches the normal distribution as sample size increase
C) has more area in the tails than normal distribution
D) all the above

Answers

According to the T distribution, the correct option is (d) All the above

T distribution:

In statistics, t-distribution is used in all forms of t-tests. Where the T-tests are all classified as inferential statistical tests and will follow all of the same steps required to complete the process of null hypothesis significance testing. And it defines the exact shape of the t-distribution is governed by the degrees of freedom. And the t-distribution is associated with a degrees of freedom equal to the sample size minus a value of 1.

Given,

Here we need to find the correct option according to the T distribution.

As per the given options, the correct answer is best represented by option D. All of the above are correct.

Because, the t-distribution, along with the normal distribution, both assume data is normally distributed in the population.

When the sample size increases towards 30, the t-distribution will resemble the normal distribution, provided the data in the population is normally distributed.

If it is correct to state that the tails of the t-distribution are thicker compared to the normal distribution.

To know more about T distribution here.

https://brainly.com/question/13574945

#SPJ4

Use a parametrization to find the flux
\int \int_{s} F.n . d\sigmaof F =5zk across the portion of the sphere x2+y2+z2=a2 where Z is positive in direction away from the origin.
the flux is____

Answers

The flux of the given sphere as calculated from the data is, flux= 5πa³/6.

We can use spherical coordinates for this sphere with radius r=ar =a.

The surface can then be described using the parametric equation as

r(θ,Ф)= acosθsinФi + asinθsinФj + acossθk

The partial derivatives of this parametric function with respect to its parameters are,

r ϕ​=acosθcosϕi+asinθcosϕj−asinϕk

r θ=−asinθsinϕi+acosθsinϕj

Therefore,

the cross product will be ,

ndσ=a²[cosθsin²ϕi+sinθsin²ϕj+sinϕcosϕk]dϕd

In these coordinates

F = 5zk = 5acosФk

therefore on integrating we get ,

flux= 5πa³/6

Flux is a physics term that refers to the number of electric or magnetic field lines that pass through a surface in a given amount of time. Field lines provide a visual representation of the magnitude and direction of the field being measured.

To learn more about flux

brainly.com/question/17052775

#SPJ4

​Virginia placed 30 marbles in a bag. Seven of the marbles are yellow, 4 marbles are orange, 5 marbles are blue, 8 marbles are green, and 6 marbles are red. Click and drag the letters onto the probability scale to order the likelihood that the event will happen.

Answers

The probabilities of each marble is given as follows:

Yellow: 7/30.Orange: 4/30.Blue: 5/30.Green: 8/30.Red: 6/30.

Hence the order of the likelihoods, from least to greatest, is given as follows:

Orange, Blue, Red, Yellow, Green.

How to calculate the probabilities?

In the context of a problem, a probability is calculated as the division of the number of desired outcomes divided by the number of total outcomes.

There are 30 marbles in this problem, hence the probability of each marble is calculated as the division of the amounts of each marble by 30.

As all the fractions have a denominator of 30, they are ordered from the lowest numerator, as follows:

Orange, Blue, Red, Yellow, Green. (from the numerator of 4 to the numerator of 8).

More can be learned about probabilities at brainly.com/question/14398287

#SPJ1

Does the relationship show a ratio or a rate explain

Answers

The relationship shows a ratio.

What is a relationship?

A relation in mathematics describes the connection between two sets of values for ordered pairs. The set of items in the first set are referred to as domain, and they are connected to the set of elements in the second set, referred to as range.

How can one tell whether a relationship is a function?

A function is described as having one and only one output for each input value. In this case, the output values are referred to as the range and the input values as the domain.

In the rate the unit of two variables are different. In the ratio the unit of the variable is the same.

To learn more about ratio, click on below link:

https://brainly.com/question/19596186

#SPJ1

the dog days clothing company receives 630 cases of imported zippers every 18 days. the number of cases of zippers on inventory t days after the shipment arrives is n(t)

Answers

The average daily inventory is 253 cases

In this question, we have been given we have been given the number of cases of zippers on inventory t days after the shipment arrives is

N(t) = 630 − 30√18t

We need to find the average daily inventory.

We use the definite integral to compute for the average value over that interval from a to b.

(∫[a_b] f(x) dx )/ (b - a)

Here, a = 1 and b = 18

so, the average daily inventory would be,

∫[18_1] N(t)dt / (18 - 1)

= ∫[18_1] (630 − 30√18t) dt / 17

= 4314.85 / 17

= 253.81

= 253

Therefore,  the average daily inventory = 253 cases

Learn more about the definite integral here:

https://brainly.com/question/29685762

#SPJ4

The complete question is:

the dog days clothing company receives 630 cases of imported zippers every 18 days. the number of cases of zippers on inventory t days after the shipment arrives is  N(t) = 630 − 30√18t. Find the average daily inventory.

Please help!! ASAP!!

Answers

Answer:

3

Step-by-step explanation:

Figure A is a dilation of Figure B by a factor of 3

That means that Figure A is 3 times larger than Figure B.

From Figure B to Figure A, the scale factor is 3.

Which of the following statement is true about triangle inequalities?

Answers

The statement which is true about triangle inequalities is that the sum of the lengths of the 2 sides of a triangle is equal than the third side of the triangle.

What are triangle inequalities?

In Euclidean geometry, triangle inequalities are a theorem which states that the sum of any two sides of a triangle is greater than or equal to the third side; in symbols, a + b ≥ c. In shorts, the theorem states that the shortest distance between two points is a straight line.

If the length of a side is equal to the sum of the other two sides, then the vertices have to be colinear, meaning that the three sides form a line segment rather than a triangle. This is called the triangle inequality.

Learn more about triangles at: https://brainly.com/question/2773823

#SPJ4

T/F show that regardless of the testing point used we arrive at the same answer for the direction of a phse plane

Answers

True, Regardless of the testing point used we arrive at the same answer for the direction of a phase plane.

We were able to sketch the solution, y(t), in the y-t plane for the single differential equation case and observe actual solutions. Due to the fact that our answers are actually vectors, this would be fairly challenging in this situation.

Here, we're going to plot the points that we represent as the system's solutions as points in the x1x2 plane. Our equilibrium solution will line up with the x1x2 plane's origin, which is also known as the phase plane.

To learn more about phase plane here:

https://brainly.com/question/15276883

#SPJ4

Which of the following is equidistant from the vertices of a triangle? a.circumference b. centroid c.orthocentre d. incentre

Answers

(Option D. Incentre) The incentre of a triangle is the point equidistant from all three of the triangle's vertices. This means that its distance from each vertex is the same.

Which of the following is equidistant from the vertices of a triangle?

Option D. Incentre.

The incentre of a triangle is an important point of reference for geometry. It is located at the center of the triangle's inscribed circle, which is the circle that touches each of the triangle's three vertices. This is why the incentre is also known as the triangle's inner center or center of the triangle's inscribed circle. As it is equidistant from all three of the triangle's vertices, the incentre is useful for finding the angles of the triangle and for finding the lengths of the triangle's sides. It can also be used to calculate the area of the triangle.

Learn more about the Triangles: https://brainly.com/question/17335144

#SPJ4

Determine whether the sequence converges or diverges. If it converges, find the limit. (If an answer does not exist, enter DNE.)
an= 1-(0.6)^n lim n tends to infinite an=n^3/7n^3+1lim n tends to infinite an=an= 7+2n^2/n+7n^2 lim n tends to infinite an=n^3/5n+8 lim n tends to infinite an= an=e^6/n lim n tends to infinite an= Square root n+5/25n+5 lim n tends to infinite an=n^2 /square root n^3+9n lim n tends to infinite an=

Answers

The given series diverges.

Given:

aₙ = n²/√n³+7n

using series divergence test

If limₙ→∞ aₙ ≠0, then ∑  aₙ diverges .

limₙ→∞ (n²/√n³+7n) = limₙ→∞ (n²/√n³(1+7/n²)

= limₙ→∞ (n²/n³/²√1+7/n²)

= limₙ→∞ (√n/√1+7/n²)

= (∞/√1+0)

= ∞

Therefore the series diverges.

Learn more about Divergence here:

brainly.com/question/17177764

#SPJ4

Between 11 p.m. and midnight on Thursday night, Mystery Pizza gets an average of 4.2 telephone orders per hour. (a) Find the median waiting time until the next telephone order. (b) Find the upper quartile of waiting time before the next telephone order. (c) What is the upper 10 percent of waiting time until the next telephone order? Show all calculations clearly.

Answers

a) The median waiting time until the next telephone order is of: 2.91 minutes.

b) The upper quartile of waiting time before the next telephone order is of: 5.82 minutes.

c) The upper 10% of waiting time until the next telephone order is of: 9.67 minutes.

What is the exponential distribution?

The exponential probability distribution, with mean m, is described by the following equation:

[tex]f(x) = \mu e^{-\mu x}[/tex]

In which [tex]\mu = \frac{1}{m}[/tex] is the decay parameter.

The mean for this problem is of:

m = 4.2.

Hence the decay parameter is of:

[tex]\mu = \frac{1}{4.2} = 0.2381[/tex]

The mass function is then given as follows:

[tex]P(X \leq x) = 1 - e^{-\mu x}[/tex]

Hence:

[tex]P(X \leq x) = 1 - e^{-0.2381x}[/tex]

The median time is the value of x for which P(X < x) = 0.5, hence:

[tex]0.5 = 1 - e^{-0.2381x}[/tex]

[tex]e^{-0.2381x} = 0.5[/tex]

[tex]\ln{e^{-0.2381x}} = \ln{0.5}[/tex]

-0.2381x = ln(0.5)

x = -ln(0.5)/0.2381

x = 2.91 minutes.

The upper quartile is the value of x for which P(X < x) = 0.75, hence:

[tex]0.75 = 1 - e^{-0.2381x}[/tex]

[tex]e^{-0.2381x} = 0.25[/tex]

[tex]\ln{e^{-0.2381x}} = \ln{0.25}[/tex]

-0.2381x = ln(0.25)

x = -ln(0.25)/0.2381

x = 5.82 minutes.

The upper 10% is the value of x for which P(X < x) = 0.9, hence:

[tex]0.9 = 1 - e^{-0.2381x}[/tex]

[tex]e^{-0.2381x} = 0.1[/tex]

[tex]\ln{e^{-0.2381x}} = \ln{0.1}[/tex]

-0.2381x = ln(0.1)

x = -ln(0.1)/0.2381

x = 9.67 minutes.

More can be learned about the exponential distribution at https://brainly.com/question/14634921

#SPJ1

Bob the trainer has two solo workout plans that he offers his clients: Plan A and Plan B. Each client does either one or the other (not both). On Monday there were 3 clients who did Plan A and 8 who did Plan B. On Tuesday there were 5 clients who did Plan A and 2 who did Plan B. Bob trained his Monday clients for a total of 15 hours and his Tuesday clients for a total of 8 hours. How long does each of the workout plans last?

Answers

The number of hours that plan A last is 1 hour and the number of hours that plan B last is 1 1/2 hours.

How long does each workout last?

The first step is to form a system of equations using the information in the question:

3a + 8b = 15  equation 1

5a + 2b = 8 equation 2

Where:

a = length of plan A b = length of plan B

The elimination method would be used to determine the required values

In order to determine the value of a, multiply equation 2 by 4

20a + 8b = 32 equation 3

Subtract equation 1 from equation 3 :

17a = 17

Divide both sides of the equation by 17

a = 17/17

a = 1 hour

Substitute for a in equation 1 :

3(1) + 8b = 15

3 + 8b = 15

8b = 15 - 3

8b = 12

b = 12 / 8

b = 1 4/8 = 1 1/2 hours

To learn more about system of equations, please check: https://brainly.com/question/25875552

#SPJ1

A bag has four balls labeled A, B, C, and D. One ball will be randomly picked, and its letter will be recorded as the outcome. Give the sample space describing all possible outcomes. Then give all of the outcomes for the event of choosing the letter A. If there is more than one element in the set, separate them with commas.

Answers

Answer:

B

hope you like it

Cos(75) using cos(a+b)

Answers

Answer:

[tex]\sqrt{6}[/tex]/4 – [tex]\sqrt{2}[/tex]/4  

Step-by-step explanation:

cos(a + b) = cos (a) cos(b) – sin (a) sin (b)

cos(75) =

cos(45 + 30) =  ==> break it down into more solvable angles

cos (45) cos(30) – sin (45) sin (30) =

[tex]\sqrt{2}[/tex]/2 * [tex]\sqrt{3}[/tex]/2 – [tex]\sqrt{2}[/tex]/2 * 1/2 =  ==> simplify

([tex]\sqrt{2}[/tex]*[tex]\sqrt{3}[/tex])/(2*2) – ([tex]\sqrt{2}[/tex]*1)/(2*2) =[[tex]\sqrt{6}[/tex]/4 – [tex]\sqrt{2}[/tex]/4]

To learn more about complex trigonometry formulas, click here:

https://brainly.com/question/25830912

what is the probability of getting a number less than 3 and a tails?; latricia determines that the probability; which value cannot represent the probability of an event occurring?; eitan and asher are playing a game; keyless entry door lock; electric door lock; best door locks; automatic door locks

Answers

The probability of getting a tails and a number less than 3 is equal to 1/6.

As given in the question,

Probability of getting heads or tail is:

P (H) = P(T) = 1/2

The probability of getting any number on the die is:

P (1) = P (2) = P(3) = P (4) = P (5) = P (6) = 1/6

The probability of getting a tail on the coin and a number less than 3 on the die is:

P(tails and a number less than 3) = P(tails) x P( a number less than 3)

= 1/2 x [P(1) +P(2)]

= 1/2 x [1/6 +1/6]

= 1/6.

Therefore ,the required probability of getting tails and a number less than 3 is  equal to 1/6.

Learn more about probability here

https://brainly.com/question/11234923

#SPJ4

P(x)=x^3+3x^2+3x+2

find the 4 rational roots

Answers

Answer:The only rational root of x3−3x2+4x−12=0 is 3.

Step-by-step explanation:

{1,−1,2,−2,3,−3,4,−4,−6,12,−12} , if at least one root is rational.

hope this helps

Will Give Brainliest if Answered Correctly look at picture

Answers

The additional jump in inches that John must make to beat the record is

5 1/4 inches

How to find the height in inches required to break the record

The problem is solved by converting the given dimensions in ft to  inches

conversion to inches is solved using

12 inches = 1 ft

The record jump is

= 23 feet 1 3/4 in

= 23 * 12 + 1 3/4 in

= 276 + 1 3/4

= 277.75 in

John's best long jump

= 22 ft 8 1/2 in

= 22 * 12 + 8 1/2 in

= 264 + 8 1/2

= 272.5 in

John's additional jump

= record jump - John's best long jump

= 277.75 - 272.5

= 5.25 in

= 5 1/4 in

Learn more about conversion of  units at:

https://brainly.com/question/141163

#SPJ1

A realtor wants to compare the mean sales-to-appraisal ratios of residential properties sold in four neighborhoods (A, B, C, and D). Four properties are randomly selected from each neighborhood and the ratios recorded for each, s shown below. A: 12, 11, 09, 0.4 C 1.0, 1.5, 1.1, 1.3 B: 2.3, 2.1,1.9, 1.6 D0.8,1.3, 1.1, 0.7 The ressults of the analysis are Sumarized in the following ANOVA table F P-value 1818 1.0606 Neighbarhood Error Total 0001 12 4.3644 (a) (Spt) Fill in the blanks in the ANOVA table. (Show your steps for partial credits (b) (2pt) Bsed on the P-value in the ANOVA, you should rejesct) Hp then conclude thnt (reject or not (c) (2pt) The realtor decided to compare the 4 population mes by using the a multiple comparison procedure. In this case, you prefer to uTukey or Bonferroni) because (d) (1pt) Given the multiple comparison procesdr comparisons that can be made in (c), there are

Answers

The solutions are,

(a) 1.1825

(b) 0.0985

(c) Reject the null hypothesis since the test's p-value (0.001) is less than 0.05. One might draw the conclusion that there is evidence of a difference in the mean ratios for the 4 neighborhoods based on the sample data.

Given data,

A realtor wants to compare the mean sales-to-appraisal ratios of residential properties sold in four neighborhoods (A, B, C, and D). Four properties are randomly selected from each neighborhood ;

(a) The within-group sum of squares for this analysis will be

4.3644 - 3.1819 = 1.1825

Hence, within the group sum of squares for this analysis is 1.1825.

(b) There are four groups so df for within groups is 12 so the within-group mean squares for this analysis will be,

[tex]\frac{1.1825}{12}=0.0985[/tex]

(c) Since the p-value of the test (0.001) is less than 0.05, reject the null hypothesis. That is based on sample evidence one can conclude that there is evidence of a difference in the mean ratios for the 4 neighborhoods.

To learn more about mean click here:

brainly.com/question/23631778

#SPJ4

This ques Part A It takes Ernest's grandfather clock 2 seconds to chime. What equation can be used to model the total time y for number of chimes x? Write the equation in the form y = kx?​

Answers

The equation that can be used to model the total time y for number of chimes is y = 2x.

What equation can be used to model the total time y for number of chimes x?

An equation is the statement that illustrates the variables given. In this case, two or more components are taken into consideration to describe the scenario.

From the information, it takes Ernest's grandfather clock 2 seconds to chime.

Therefore, the equation that can be used to model the total time y for number of chimes x will be:

y = 2 × x

y = 2x.

The equation is y = 2x.

Learn more about equations on:

brainly.com/question/2972832

#SPJ1

write a statement that associates s with a set that contains the following elements: 23, 42, -11, 89.

Answers

A statement that associates s with a set that contains the following elements: 23, 42, -11, 89 is s = set ( [ 23 , 42 , -11 , 89 ] )

Given :

a statement that associates s with a set that contains the following elements: 23, 42, -11, 89.

Statement :

A statement is an instruction that the Python interpreter can execute.

We have seen two kinds of statements: print and assignment.

When you type a statement on the command line, Python executes it and displays the result, if there is one. The result of a print statement is a value.

we can write :

s = set ( [ 23 , 42 , -11 , 89 ] )

learn more about the set here:

https://brainly.com/question/8053622

#SPJ4

185

+50%+4%=start fraction, 185, divided by, 100, end fraction, plus, 50, percent, plus, 4, percent, equals

Answers

Answer:

1.85

Step-by-step explanation:

185/100 equals 1.85

if I'm reading the problem correctly this is what we have:

0.54 = 1.85 + 0.54

for the left side to equal the right side you must add 1.85 or 185/100

A survey determines that six out of every ten Niceville residents shop at Walmart: In a group of15, randomly selected Nicevillians, find the prabablllythat at least thirteenof them shop at Walmart.

Answers

From a group of 15 randomly selected residents , the probability that at least thirteen of them shop at Walmart is 0.0271  .

6 out of 10 Niceville residents shop at Walmart

So, the probability of shopping at Walmart (p) is = 6/10 = 0.6

The probability of not shopping at Walmart is (q) = 1 - 0.6 = 0.4 .

The number of members in the group is(n) = 15 ;

We have to find the probability that, at least thirteen of them shop at Walmart

By the Binomial Probability Distribution,

we know that ; P(x = a) = ⁿCₐ×pᵃ×qⁿ⁻ᵃ

So , P(x ≥ 13) = P(x=13) + P(x=14) + P(x=15)

Substituting the values of a , n and p and q ,

We get ;

= ¹⁵C₁₃p¹³q² + ¹⁵C₁₄p¹⁴q¹ + ¹⁵C₁₅p¹⁵q⁰

= ¹⁵C₁₃(0.6)¹³(0.4)² + ¹⁵C₁₄(0.6)¹⁴(0.4)¹ + ¹⁵C₁₅(0.6)¹⁵(0.4)⁰

= 0.0219 + 0.0047 + 0.0004

= 0.0271

Therefore, the required probability is 0.0271

Learn more about Binomial Probability Distribution here

https://brainly.com/question/26077592

#SPJ4

Write 7 as the ratio of two integers.(Enter a ratio in fraction form.)

Answers

Answer:

14/2

Step-by-step explanation:

We can write 7 as 7/1

Multiply the top and bottom by 2/2

7/1 * 2/2 = 14/2

We could multiply by any number over itself

7/1 * 3/3 = 21/3

This is an equivalent fraction

Answer:

See below

Step-by-step explanation:

Examples ( choose one)

14/2

7/1

21/3

98/14

1 out of 35 students is going on a field trip. What is the ratio of the number of students who are staying at school to the number of students who are going on the field trip?

Answers

Answer:

35:1 because that is the most likely answer

6. The following polygon is a regular octagon with
center O. What is the value of x?
to

Answers

Check the picture below.

what ratio, a:b, is represented in the table?

a b

1.5 2.5

51 85

99 165

Answers

A 4:54:5

B 2:32:3

C 3:5

Answers

Answer:is c or 3:5

Step-by-step explanation:

Other Questions
Question 22 Cr is a member of which family? a. noble gases b. halogens c. alkaline earth metals d.alkaline metals e.None of these 2. The following tatement decribe a trong intenity of an earthquake, EXCEPT?a. Generally felt by mot people indoor and outdoorb. Dizzine and nauea are experienced by ome peoplec. Many leeping people are awakenedd. Strong haking and rocking felt throughout building Determine between which consecutive integers the real zeros of f(x)=x-4x-2 are located.a. between 3&4 and -1&0c. between 4&5 and -1&0between 3&4 and 0&1b. between 4&5 and 0&1d.Please select the best answer from the choices provided social facilitation performance for well-learned tasks and performance for simple tasks. jacob, a researcher, asks the participants of his study to transfer data from 12 spreadsheets to 12 different documents. he observes that the participants gain more efficiency in transferring the data with each new spreadsheet. this scenario exemplifies a(n) Tell whether the table of values represents a linear, exponential, or quadratic function. According to the passage,what were the causes of World War II in Europe?O A. Germany's refusal to honor prewar agreements and Japan's attackon Pearl Harbor B. Japan's attack on the United States and the capture of all GermanterritoryC. The German invasion of Poland and the United States' use ofatomic bombs against JapanD. The French and British appeasement policies and the Germaninvasion of Polans The total cost of the snow cones, t, depends on the number of snow cones ordered, s. The relationship is shown in the table.Number ofSnow Cones Total Cost(dollars)2 64 125 15Which equation can be used to calculate the total cost, t, of s snow cones? you now know that inheritance of eye color in fruit flies is sex-linked: the gene encoding eye color is located on the x chromosome, and there is no corresponding gene on the y chromosome. how would the inheritance pattern differ if the gene for eye color were instead located on an autosome (a non-sex chromosome)? recall that for autosomes, both chromosomes of a homologous pair carry the same genes in the same locations.suppose that a geneticist crossed a large number of white-eyed females with red-eyed males. a cross of two flies where the female fly has white eyes, and the male fly has red eyes.consider two separate cases:case 1: eye color exhibits sex-linked inheritance.case 2: eye color exhibits autosomal (non-sex-linked) inheritance. (note: in this case, assume that the red-eyed males are homozygous.)for each case, predict how many of the male and female offspring would have red eyes and white eyes.drag the correct numbers on the left to complete the sentences on the right. numbers can be used once, more than once, or not at all. under u.s. laws, who has the right to hold an election to choose what union they want to represent them, if any? multiple choice only private workers only public workers most private and public workers only the employer before selling bonds to investors, matteo's company must provide audited financial statements and a detailed description of the terms of the bonds. by doing so, which federal regulation is he complying with? suppose the united states wants to open its trade borders to the world market for coffee. which of the following will determine whether the world price for coffee is affected? assume that the salaries of fast food restaurant managers are normally distributed with a mean of $48,000 and a standard deviation of $4,000. what is the lowest amount that a manager needs to make and still be one of the top 7% earners? use excel, and round your answer to the nearest dollar. provide your answer below: Why did the Suez Canal increased European interest?; What was the purpose of Suez Canal?; What impact did the Suez Canal have?; How did the Suez Canal change the world? true or false? buyer personas are non-fictional case study examples of your most successful customer. on which continent could you potentially see live colonies of all extant honey bee species? One barrier to problem solving is the tendency to impose constraints that dont exist. a. trueb. false according to self-discrepancy theory, what emotion is most likely to arise from discrepancies between our ought and actual selves? the northern leopard frog, tree frog, and wood frog are all different species of frogs. why is having a scientific name for each species of an organism important for scientists? rna polymerase binds to a ________ to initiate the process of ________.